1
$\begingroup$

enter image description here

Attempt:

First, I call $x$ be the number of shafts produced per year and $y$ the number of frames produced per year. We have that each machine works at most $4500$ hours. we can place all of our data in the following table:

enter image description here

Thus, we want to maximize $f(x,y) = x+y$ subject to

$$ 0.6 x + 0.8 y \leq 90 000 $$ $$ 0.3 x + 0 y \leq 22 500 $$ $$ 0.4 x + 0.6 y \leq 45 000 $$ $$ 0x + 0.2 y \leq 135 00 $$ $$ 0x + 0.3 y \leq 27 000 $$

and obviously $x \geq 0$ and $y \geq 0 $

Is this the correct formulation?

$\endgroup$
4
  • $\begingroup$ Can a forging machine process a shaft and a frame at the same time? $\endgroup$ Commented Sep 3, 2018 at 22:02
  • $\begingroup$ no, I dont think so. $\endgroup$ Commented Sep 3, 2018 at 23:18
  • $\begingroup$ Since fractional shafts and frames are probably not desired, you should add $x \in \mathbb{Z}$ and $y \in \mathbb{Z}$. This, however, turns your LP into a MIP. $\endgroup$ Commented Sep 4, 2018 at 6:39
  • $\begingroup$ What is an MIP? But, as far as the modeling is concerned, am I correct? $\endgroup$ Commented Sep 4, 2018 at 6:44

1 Answer 1

2
$\begingroup$

I suppose your model is not correct. Solving the model with an LP-solver yields the solution $x=75000$ and $y=25000$. I assume, however, that for every frame you also need a shaft. Hence, you need to add the following constraint:

$$x=y$$

Solving the problem with the additional constraint yields the solution $x=y=45000$. Please note that luckily the number of shafts/frames in the solution is an integer. This need not happen always. Thus, you should also add the constraints $x\in \mathbb{Z}$ and $y \in \mathbb{Z}$ in general, even though these were not needed here.

$\endgroup$
4
  • $\begingroup$ the constraint is $x \geq y$ (not $x=y$) $\endgroup$ Commented Sep 6, 2018 at 17:41
  • $\begingroup$ @LinAlg Could you please elaborate on this? $\endgroup$ Commented Sep 6, 2018 at 17:44
  • $\begingroup$ The requirement "for every frame you also need a shaft" means that the number of shafts needs to be at least the number of frames (so $x \geq y$). Your solution probably remains optimal though. $\endgroup$ Commented Sep 6, 2018 at 18:07
  • $\begingroup$ @LinAlg In the original problem it did not say "for every frame you also need a shaft" but that the assembly consists "of a frame, a shaft, and a ball bearing". The other quote was by me and was due to that in the author's own problem formulation one would have obtained a higher number of frames than shafts. I still think that my constraint is correct in terms of the text in the author's post. It is just my wording that caused the confusion. $\endgroup$ Commented Sep 7, 2018 at 6:58

You must log in to answer this question.

Start asking to get answers

Find the answer to your question by asking.

Ask question

Explore related questions

See similar questions with these tags.